Answer:48 outfits.
Step-by-step explanation:
Number of outfits that Juan can wear
N =number of ties x number of pants x number of pants
N =4 x 3 x 4
N =48 outfits
slope -1/2, passes through (0, 5)
Our standard linear slope equation is:
mx + b = y
We're told our slope is 1/2, so that's our m value.
When x = 0 our value is 5. If you need this visualized, then:
1/2 * 0 +b = 5
0 + b = 5
b = 5
So our equation looks like:
x/2 + 5 = y
At the beginning of spring, Samantha planted a small sunflower in her backyard. When it was first planted, the sunflower was 20 inches tall. The sunflower then began to grow at a rate of 1 inch per week. How tall would the sunflower be after 4 weeks? How tall would the sunflower be after w weeks?
Answer:
24 inches
Step-by-step explanation:
we assume that with a linear relationship between two variables, for any fixed value of x, the observed ________ follows a normal distribution.
We assume that with a linear relationship between two variables, for any fixed value of x, the observed residuals follows a normal distribution.
This assumption is based on the Central Limit Theorem, which states that when the sample size is large enough, the distribution of sample means will be approximately normal, regardless of the shape of the underlying population distribution.
In the case of a linear relationship between two variables, we can assume that the residuals (the difference between the observed y values and the predicted values based on the linear regression model) follow a normal distribution with mean 0 and constant variance. This assumption is important because it allows us to use statistical methods that rely on normality, such as hypothesis testing and confidence intervals.
Learn more about Normal distribution:
https://brainly.com/question/1846009
#SPJ11
A number times nine + four gives you fifteen if you have time
Answer:
11 / 9
Step-by-step explanation:
Let number = x
(x * 9) + 4 = 15
9x + 4 = 15
9x = 15 - 4
9x = 11
x = 11 / 9
Hence, the number is 11 /9
James wants to tile his floor using tiles in the shape of a trapezoid to make the pattern a little more interesting he has decided to cut the tiles in half along the median to top base of each time is 12 inches in length and turn bottom base is 16 inches
Answer: 14 inches
Step-by-step explanation:
Given the following :
James wants to cut a floor tile in the shape of a trapezium with the following dimension :
Top base = 12 inches
Bottom base = 16 inches
To obtain the medan, we take the average of the two bases :
(top base + bottom base) / 2
(12 inches + 16 inches) / 2
28 inches / 2
= 14 inches.
Divide. Express your answer in simplest form.
2 1/2 ÷ 2 1/6
Answer:
15/13 OR 1.154
Step-by-step explanation:
2 1/2=5/2
2 1/6=13/6
5/2 / 13/6
=5/2 x 6/13
=30/26
=15/13
=1.15384615385
≈ 1.154
How many degrees is the exterior angle of a triangle (w) if the non-adjacent angles (z and y) are 46 and 85?
Answer:
46
Step-by-step explanation:
WILL GIVE BRAINLIEST IF RIGHT IG....PLEASE get it RIGHT....
find the volume of a cone with a diameter of 12m and a height of 8m.
ANSWER CHOICES
oA. 96m^3
oB. 384 pi m^3
oC. 96 pi m^3
oD. 288 pi m^3
Answer:
V = 96 pi m^3
Step-by-step explanation:
V = (1/3) pi(r^2)h
Radius = 1/2(Diameter)
R = 1/2(12)
R = 6
Input.
V = (1/3) pi(6^2)8
Solve.
V = (1/3) pi(36)8
V = (1/3) 904.32
V = 301.44
V = 301.44/pi
V = 96 pi m^3
Answer:
The correct answer is option (C) 96π m³.
Step-by-step explanation:
As per given question we have provided that :
⟶ Diameter = 12 m⟶ Radius = 12/2 = 6 m⟶ Height = 8 mHere's the required formula to find the volume of cone :
\({\longrightarrow{\pmb{\sf{V_{(Cone)} = \dfrac{1}{3}\pi{r}^{2}h}}}}\)
➝ V = Volume➝ π = 22/7➝ r = radius➝ h = heightSubstituting all the given values in the formula to find the volume of cone :
\({\implies{\sf{V_{(Cone)} = \dfrac{1}{3}\pi{r}^{2}h}}}\)
\({\implies{\sf{V_{(Cone)} = \dfrac{1}{3}\pi{(6)}^{2}8}}}\)
\({\implies{\sf{V_{(Cone)} = \dfrac{1}{3}\pi{(6 \times 6)}8}}}\)
\({\implies{\sf{V_{(Cone)} = \dfrac{1}{3}\pi \times {(36)} \times 8}}}\)
\({\implies{\sf{V_{(Cone)} = \dfrac{1}{3} \times \pi \times 36 \times 8}}}\)
\({\implies{\sf{V_{(Cone)} = \dfrac{1}{\cancel{3}} \times \pi \times \cancel{36} \times 8}}}\)
\({\implies{\sf{V_{(Cone)} = \pi \times 12 \times 8}}}\)
\({\implies{\sf{V_{(Cone)} = \pi \times 96}}}\)
\({\implies{\sf{V_{(Cone)} =96\pi}}}\)
\(\star{\underline{\boxed{\sf{\red{V_{(Cone)} =96\pi \: {m}^{3}}}}}}\)
Hence, the volume of cone is 96π m³.
\(\rule{300}{2.5}\)
You purchase a rectangular piece of metal that has dimensions 5. 0 x 15. 0 x 30. 0 mm and mass 0. 0158 kg. The seller tells you that the metal is gold. To check this, you compute the average density of the piece. What value do you get? were you cheated?.
The density of metal is 7022.2 kg/m^3 and the metal is not gold. We were cheated.
In the given question, we purchase a rectangular piece of metal that has dimensions 5.0 x 15.0 x 30.0 mm and mass 0.0158 kg.
The seller tells you that the metal is gold.
We have to find the density of piece.
Then Given ,
Mass = 0.0158 kg
Volume = 5.0 mm * 15.0 mm * 30.0 mm
Density = mass / volume
Putting the values
Density = 1.58 * 10^(-2) kg/(5.0*10^{-3}*15.0*10^{-3}*30.0*10^{-3}) m^3
Density = 7022.2 kg/m^3
The density of metal is 7022.2 kg/m^3 and the metal is not gold. We were cheated.
To learn more about density of metal link is here
brainly.com/question/29009519
#SPJ4
pls help on my maths question
Answer:
Somewhat illegible but looks like the equation should be:
S = u t + 1/2 a t^2
100 = 20 + 1/2 * a * 4 where t = 2
a* 4 = 160
a = 40
PLEASE HELP!! Select all that apply, brainiest for right answer! right answers only please
Answer:
D
Step-by-step explanation:
May someone please help me with this :)
Answer:
a) 80in peri, 320 in²
b) 19.426 peri, 26.139
17. A man stands 4300 feet from the Burj Khalifa in
Dubai (world's tallest building). The angle of
elevation from his eyes (5 feet) to the top of the
building is 32°. How tall is the building?
9514 1404 393
Answer:
b. 2692 ft
Step-by-step explanation:
The relation of interest between the sides of the right triangle is ...
Tan = Opposite/Adjacent
Using the given numbers, that is ...
tan(32°) = (building height)/(4300 ft)
Multiplying by 4300 ft, we get ...
building height = (4300 ft)·tan(32°) ≈ 2686.9 ft
This is the height above the observer's eyes, so the height of the building above the ground is ...
2687 +5 = 2692 . . . . feet
The building is 2692 feet tall.
A factory sampled hundreds of 9oz bags of chips off of one of their production lines to make sure the bags had the appropriate
amount of chips. They found the amount of chips to be normally distributed with = 9.12 ounces and o= .05 ounces.
PART A
Given this information, fill out the normal distribution chart below for the weight of 9oz bags of chips from this factory.
The probability that a bag of chip is lower than 9.07 oz of weight is 0.1587.
What is Normal Distribution ?A continuous probability distribution for a real-valued random variable is called a normal distribution or a Gaussian distribution.
An example of a continuous probability distribution is the normal distribution, in which the majority of data points cluster in the middle of the range while the remaining ones taper off symmetrically toward either extreme. The distribution's mean is another name for the center of the range.
The normal distribution defines how a variable's values are distributed, just like any probability distribution does. Because it properly depicts the distribution of values for many natural occurrences, it is the most significant probability distribution in statistics.
the mean of the data = 9.12 oz
The SD = 0.05 oz
The probability of the bag to be less than 9.07 oz
x = 9.07
z = x - mean / SD = 9.07 - 9.12/0.05 = - 1
for z = -ve
Ф(z) = 1 - Ф(-z) = 1 - Ф(1) = 1 - 0.8413 = 0.1587
The probability that a bag of chip is lower than 9.07 oz of weight is 0.1587.
To learn more about Normal Distribution refer to :
https://brainly.com/question/20815863
#SPJ1
help meeeeeee.....!!!!
I WILL MARK BRAINIEST!!!
Answer:
X = 35/8
Y = -5
Describe fully the single transformation that maps a onto triangle b
The single transformation that maps a onto triangle b is a rotation of 180 degrees across the common vertex of triangles a and b
How to determine the transformation?From the figure, we have the following highlights
Both triangles are congruentBoth triangles have one common vertexBoth triangles meet at a right angle. this implies a 180-degree rotationThe above highlights mean that the single transformation that maps a onto triangle b is a rotation of 180 degrees across the common vertex of triangles a and b
Read more about transformation at:
https://brainly.com/question/4289712
#SPJ1
This is my last question !
Answer:
x=50 degrees
Step-by-step explanation:
I'm not sure but hope this helps and have a wonderful day!!!!
Answer:
x = 48
Step-by-step explanation:
2x + 10 + x + 26 = 180
3x + 36 = 180
3x = 180 - 36
3x = 144
x = 48
Find Sn for the following geometric sequences described.
From the question, the sum of each of the geometric sequence are;
1) 31 3/4
2) 340
3) 11/16
4) -6, 12, -24
What is geometric sequence?
We have that;
Sn = a(1 -\(r^n\))/1 - r
Sn = 16(1 \(- (1/2)^7\))1 - 1/2
Sn = 16(1 - 1/128)/1/2
Sn = 16(127/128) * 2
Sn = 31 3/4
2) Un = a\(r^n\) -1
256 = \(4(4)^n-1\)
64 =\(4^n-1\)
\(4^3 = 4^n-1\)
n = 4
Sn= \(4(4^4 - 1)\)/4 - 1
Sn = 340
3) Since we have a5 then n = 5
Sn = 1(1 - (\(-1/2)^5\))/1 -(-1/2)
Sn = 33/32 * 2/3
= 11/16
4) 30= a(1 -\((-2)^4\))/1 - (-2)
30 = a(-15)/3
30 = -5a
a = 30/-5
a = -6
Then the first three terms are;
-6, 12, -24
Learn more about geometric sequence:https://brainly.com/question/27852674
#SPJ1
on a necklace two-thirds of the bees are blue of the blue beads 1/4 or oval shaped what fraction of the necklace is made up of blue ovals
on a necklace two-thirds of the bees are blue of the blue beads 1/4 or oval shaped what fraction of the necklace is made up of blue ovals
we have that
2/3 ------> blue
1/4(2/3)=2/12-1/6
answer is 1/6
i^32=......................
3 over 4 = 1 over 4 m
The value of m in the proportion is given as follows m = 3.
What is a proportion?A proportion is a fraction of a total amount, and the measures are related using a rule of three.
The relations between variables, either direct or inverse proportional, can be built to find the desired measures in the problem.
The proportion is defined as follows:
3/4 = (1/4)m.
As the values have a proportional relationship, the value of m can be obtained applying cross multiplication, as follows:
3/4 = (1/4)m
cross multiplication;
3/4 x 4 = m
m = 3
Hence m =3 is the value that satisfies the proportion in this problem.
More can be learned about proportions at brainly.com/question/24372153
#SPJ1
What is the formula to find P(A) for a series of simple events (ex: tossing a coin and selecting a number at the same time)
The probability of event A is P(A) = 1/4 or 0.25.
The formula to find P(A) for a series of simple events is:
P(A) = (number of outcomes that satisfy the event A) / (total number of possible outcomes)
For example, if you are tossing a coin and selecting a number at the same time, and event A is defined as getting a head and an even number, then:
- The number of outcomes that satisfy event A is 1 (getting a head and an even number, i.e., H2)
- The total number of possible outcomes is 4 (H1, H2, T1, T2)
- Therefore, the probability of event A is P(A) = 1/4 or 0.25.
Visit to know more about Probability:-
brainly.com/question/13604758
#SPJ11
determine if each relationship represtents a fuctikn chose yes or no for each relationship
Answer:
Yes
no
no
yes
Step-by-step explanation:
I hope it's serving
:v
Use integration in cylindrical coordinates in order to compute the volume of: U={(x,y,z):0≤x2+y2≤1,0≤z≤5−x−y}
The volume of the given set using integration in cylindrical coordinates is 12π.
Main part: The given set U can be visualized as a cylinder of radius 1 and height 5, that is truncated by removing a smaller cylinder of the same height and radius 1/2. We will use cylindrical coordinates (ρ,θ,z) to describe points in space. The volume of the set U is given by the integral:
∫∫∫_U〖dV=∫_0^(2π)∫_0^1∫_0^(5-r)ρdzdρdθ
Solution: We can see that z varies from 0 to 5 - x - y. In cylindrical coordinates, x = ρ cos θ and y = ρ sin θ. Therefore, z = 5 - ρ cos θ - ρ sin θ. Since ρ goes from 0 to 1, we have:
∫_0^(2π)∫_0^1∫_0^(5-r)ρdzdρdθ = ∫_0^(2π)∫_0^1∫_0^((5-ρcosθ-ρsinθ))ρdzdρdθ
= ∫_0^(2π)∫_0^1[5ρ - ρ²/2 - ρ² cos θ/2 - ρ² sin θ/2] dρ dθ
= ∫_0^(2π)[5/2 - 1/2 cos θ - 1/2 sin θ] dθ
= 2π [5/2 - 1/2 cos θ - 1/2 sin θ] from 0 to 2π
= π (5 - cos θ - sin θ) from 0 to 2π
= π (5 - (-1) - 0 - (-1) - 0)
= 12 π
Therefore, the volume of the set U is 12 π.
Conclusion: Thus, the volume of the given set using integration in cylindrical coordinates is 12π.
To know more about volume visit
https://brainly.com/question/6071957
#SPJ11
. Find two polynomial expressions whose quotient, when simplified, is 1/x . Use that division problem to determine whether polynomials are closed under division.
Answer:
The two polynomials are:
(x + 1) and (x² + x)
Step-by-step explanation:
A polynomial is simply an expression which consists of variables & coefficients involving only the operations of addition, subtraction, multiplication, and non - negative integer exponents of variables.
Now, 1 and x are both polynomials. Thus; 1/x is already a quotient of a polynomial.
Now, to get two polynomial expressions whose quotient, when simplified, is 1/x, we will just multiply the numerator and denominator by the same polynomial to get more quotients.
So,
Let's multiply both numerator and denominator by (x + 1) to get;
(x + 1)/(x(x + 1))
This gives; (x + 1)/(x² + x)
Now, 1 and x are both polynomials but the expression "1/x" is not a polynomial but a quotient and thus polynomials are not closed under division.
Which is the approximate measure of angle jgh? 18.4° 19.5° 70.5° 71.6°
The approximate measure of angle JGH is 70.5°.
According to this question, we have a right triangle in which lengths of its hypotenuse (GH), in centimeters, and one leg (JH) are known and we must determine the measure of an angle (∠JGH), in degrees. A representation of this triangle is included in the image attached below.
By trigonometry we have the following expression for the required angle:
cos∠GJH = HJ/GH .......(1)
Given, HJ = 2, GH = 6
cos∠GJH = 2/6
∠GJH = \(cos^{-1}\frac{2}{6}\)
≈ 70.529
The approximate measure of angle JGH is 70.5°.
To learn more about angle here:
https://brainly.com/question/28451077
#SPJ4
Let X
=
A
.
¯¯¯¯¯¯
B
C
. Evaluate X for
(a) A
=
1
,
B
=
0
,
C
=
1
, (b) A = B = C = 1 and ( c) A = B = C = 0.
The given expressions, when A=1, B=0, and C=1, X evaluates to 1.001; when A=B=C=1, X evaluates to 1.111; and when A=B=C=0, X evaluates to 0.000. These evaluations are based on the given values of A, B, and C, and the notation ¯¯¯¯¯¯BC represents the complement of BC.
To evaluate the expression X = A.¯¯¯¯¯¯BC, we substitute the given values of A, B, and C into the expression.
(a) For A = 1, B = 0, and C = 1:
X = 1.¯¯¯¯¯¯01
To find the complement of BC, we replace B = 0 and C = 1 with their complements:
X = 1.¯¯¯¯¯¯01 = 1.¯¯¯¯¯¯00 = 1.001
(b) For A = B = C = 1:
X = 1.¯¯¯¯¯¯11
Similarly, we find the complement of BC by replacing B = 1 and C = 1 with their complements:
X = 1.¯¯¯¯¯¯11 = 1.¯¯¯¯¯¯00 = 1.111
(c) For A = B = C = 0:
X = 0.¯¯¯¯¯¯00
Again, we find the complement of BC by replacing B = 0 and C = 0 with their complements:
X = 0.¯¯¯¯¯¯00 = 0.¯¯¯¯¯¯11 = 0.000
In conclusion, when A = 1, B = 0, and C = 1, X evaluates to 1.001. When A = B = C = 1, X evaluates to 1.111. And when A = B = C = 0, X evaluates to 0.000. The evaluation of X is based on substituting the given values into the expression A.¯¯¯¯¯¯BC and finding the complement of BC in each case.
Learn more about expressions here:
brainly.com/question/13838432
#SPJ11
what is 8 divided by 7.6
Answer:
1.05263
explanation:
8 divided by 7.6
\(\hookrightarrow \dfrac{8}{7.6}\)
multiply both by 10
\(\hookrightarrow \dfrac{80}{76}\)
final answer
\(\hookrightarrow 1.05263\)
Please help me, due soon, i will give brainliest!! pre-calc question: according to the intermediate value theorem, in what interval must cos x= 2^x-6 have a solution? show work!
According to the Intermediate Value Theorem, if a function f(x) is continuous on a closed interval [a, b], and if f(a) and f(b) have opposite signs, then there exists at least one value c in the interval (a, b) such that f(c) = 0. According to the Intermediate Value Theorem, the interval in which the equation cos(x) = 2^(x-6) must have a solution is (0, π/2).
Let's apply this theorem to the function f(x) = cos(x) - 2^(x-6).
First, we need to find two values a and b such that f(a) and f(b) have opposite signs. Let's start with a = 0 and b = π/2.
f(0) = cos(0) - 2^(0-6) = 1 - 1/64 > 0
f(π/2) = cos(π/2) - 2^(π/2-6) = 0 - 1/4 < 0
Since f(a) and f(b) have opposite signs, there exists at least one value c in the interval (0, π/2) such that f(c) = 0. Therefore, the equation cos(x) = 2^(x-6) has at least one solution in the interval (0, π/2).
To summarize, according to the Intermediate Value Theorem, the interval in which the equation cos(x) = 2^(x-6) must have a solution is (0, π/2).
To knnow more about Theorem visit:
https://brainly.com/question/28667736
#SPJ11
There are 100 cm in a meter. write an expression to find the number of centimeters in 8 meters.
Help me with this question plss
Answer:
800m
Step-by-step explanation:
If 100cm is equivalent to a meter (1 Meter). 8 Meters would be equivalent to 8 Meters multiplied by 100cm and divided by a meter (1 Meter)